Finding $\angle MBC$ in Triangle $ABC$ with Proof

In summary, Ceva's theorem tells us the product of the ratios of the pairs of segments formed on each side of a triangle is equal to the sum of the products of the ratios of the segments formed on each side of the triangle.
  • #1
anemone
Gold Member
MHB
POTW Director
3,883
115
Given a triangle $ABC$ such that $\angle BAC=103^{\circ}$ and $\angle ABC=51^{\circ}$. Let $M$ be a point inside triangle $ABC$ such that $\angle MAC=30^{\circ}$, $\angle MCA=13^{\circ}$.

Find $\angle MBC$ with proof.
 
Mathematics news on Phys.org
  • #2
anemone said:
Given a triangle $ABC$ such that $\angle BAC=103^{\circ}$ and $\angle ABC=51^{\circ}$. Let $M$ be a point inside triangle $ABC$ such that $\angle MAC=30^{\circ}$, $\angle MCA=13^{\circ}$.

Find $\angle MBC$ with proof.
my solution:
View attachment 1817
from the sketch it is easy to prove that $\angle MBC=17^o=y=\dfrac {\angle ABC}{3}$
 

Attachments

  • Angle MBC.jpg
    Angle MBC.jpg
    12.7 KB · Views: 83
  • #3
Albert said:
my solution:
View attachment 1817
from the sketch it is easy to prove that $\angle MBC=17^o=y=\dfrac {\angle ABC}{3}$
Hello.

Could you show it?.

I have managed to find out, only:

[tex]\overline{OC}=[/tex], angle bisector of [tex]\angle ACB[/tex]

certain angles: 13, 26, 30, 34, 43, 51, 56, 60, 64, 73, 77, 103, 116, 129.

Regards.

Edit: I'm sorry. I had not seen, before, that it was already drawing.:eek:
 
  • #4
[sp]
Albert said:
my solution:

View attachment 1817
from the sketch it is easy to prove that $\angle MBC=17^o=y=\dfrac {\angle ABC}{3}$
In case there are other people like me who took a while to follow Albert's clever proof, here is how it works. You are given the angles of triangle ABC: $103^\circ$ at A, $51^\circ$ at B and therefore $26^\circ$ at C. You are also given the angles of triangle AMC: $30^\circ$ at A, $13^\circ$ at C and therefore $137^\circ$ at M. Since 13 is half of 26 it follows that MC bisects the angle at C. So if we drop perpendiculars E, F from M to AC and BC, the right-angled triangles MEC, MFC will be congruent, with angle $77^\circ$ at M. Now draw the circle centred at M touching AC, BC at E and F, and let BG be the other tangent to the circle from B. Then the right-angled triangles BFM, BGM are congruent. At this stage we know the angles $60^\circ$, $77^\circ$, $77^\circ$ at M. So the remaining angle AMF is $146^\circ$ and this is bisected by BM. Therefore the angle BMF is $73^\circ$; and its complement, the angle MBC, is $17^\circ.$[/sp]
 
  • #5
Opalg, maybe you can enlighten me about Albert's proof. I fail to see why G, the intersection of AM with the circle, must be a tangent point from B. I had played with the problem before looking at the solution and came up with the following:

View attachment 1821
So an indirect proof follows by showing the given construction must be the triangles as given in the problem. I think this is right.
 

Attachments

  • MHBgeometry7a.png
    MHBgeometry7a.png
    13.4 KB · Views: 72
Last edited:
  • #6
johng said:
Opalg, maybe you can enlighten me about Albert's proof. I fail to see why G, the intersection of AM with the circle, must be a tangent point from B...
That is a very good point, which I had completely overlooked. The only way round it that I can see is to do essentially as you suggest. Namely, suppose that the tangent at G meets the line BC at some point B', and then show that B'=B.
 
  • #7
MG is a radius , so $\angle BGM=90^o$

MG is a tangent line of circle M

here point M is the incenter of triangle BHC
 
Last edited:
  • #8
Albert said:
MG is a radius , so $\angle BGM=90^o$

MG is a tangent line of circle M

here point M is the incenter of triangle BHC
That implies that you are defining G as the point where the tangent from B meets the circle. In that case, you need to explain why G lies on the line AM.
 
Last edited:
  • #9
$\angle FBG=\dfrac{77+77+60-146}{2}=34^o$

$\therefore \angle GBA=17^o$

and $\angle AGB=180-73-17=90^o$
 
  • #10
Albert said:
$\angle FBG=\dfrac{77+77+60-146}{2}=34^o$

$\therefore \angle GBA=17^o$

and $\angle AGB=180-73-17=90^o$
Yes, that seems to work. You are saying that the angles BAG and BAM are both $73^\circ$. So G (defined as the point where the tangent from B meets the circle) lies on the line AM.

I think this discussion shows that these "proofs by picture" are inadequate on their own. Unless they are accompanied by a careful explanation of how each constructed point is defined, together with the order in which deductions are made, the structure of the proof is almost impenetrable.
 
  • #11
Albert, Opalg, someone please help. I still don't get it. In Albert's expression for angle BFG, the 77+77+60 is the angle AME, but from where comes 146?? Is he saying triangle AMB is isosceles?

The following sketch shows G' as the second tangency point from B and G the intersection of AM with the circle. I still fail to see why G = G'. (Of course the drawing is not accurate; the actual problem has G = G'.)

View attachment 1826
 

Attachments

  • MHBgeometry7b.png
    MHBgeometry7b.png
    9.4 KB · Views: 66
  • #12
146=360-77-77-60
 
  • #13
Albert said:
146=360-77-77-60

Yes, but the problem is that prove that it are those angles and not others
 
  • #14
First off, my first thank goes to Albert, for you provided a proof with diagram in this challenge problem. But, like others, I have a really hard time to digest your proof and I must look at the follow-up post by Opalg to fully decipher the reasoning behind your proof. Still, I am struggling to see why there must be true that the line $BGH$ is a tangent to the circle at the point $G$. I am sorry...

I want to also thank to Opalg, for your willingness to answer to johng's posts, because I always think it's the OP's responsibility to answer to anyone who has found the proof confounding...

My solution:

View attachment 1834
Ceva's theorem tells us the product of the ratios of the pairs of segments formed on each side of the triangle by the intersection point is equal to 1.

Hence we have

$\dfrac{\sin 13^{\circ}}{\sin 13^{\circ}} \cdot \dfrac{\sin 73^{\circ}}{\sin 30^{\circ}} \cdot \dfrac{\sin x}{\sin (51^{\circ}-x)}=1$

$\sin 73^{\circ}\cdot \sin x=\sin 30^{\circ} \cdot \sin (51^{\circ}-x)$

$\sin 73^{\circ}\cdot \sin x=\sin 30^{\circ} \cdot (\sin 51^{\circ} \cos x-\cos 51^{\circ} \sin x)$

$\sin x(\sin 73^{\circ}+\sin 30^{\circ}\cos 51^{\circ})=\sin 30^{\circ} \sin 51^{\circ} \cos x $

$\begin{align*}\tan x&=\dfrac{\sin 30^{\circ} \sin 51^{\circ}}{\sin 73^{\circ}+\sin 30^{\circ}\cos 51^{\circ}}\\&=\dfrac{\dfrac{1}{2} \sin 51^{\circ}}{\sin 73^{\circ}+\dfrac{1}{2}\cos 51^{\circ}}\\&=\dfrac{\sin 51^{\circ}}{2\sin 73^{\circ}+\cos 51^{\circ}}\\&=\dfrac{\sin 3(17)^{\circ}}{2\sin (90-17)^{\circ}+\cos 3(17)^{\circ}}\\&=\dfrac{\sin 17^{\circ}(3-4\sin^2 17^{\circ})}{2\cos 17^{\circ}+\cos 17^{\circ}(4\cos^2 17^{\circ}-3)}\\&=\dfrac{\sin 17^{\circ}(3-4\sin^2 17^{\circ})}{\cos 17^{\circ}(2+4\cos^2 17^{\circ}-3)}\\&=\dfrac{\sin 17^{\circ}(3-4\sin^2 17^{\circ})}{\cos 17^{\circ}(4(1-\sin^2 17^{\circ})-1)}\\&=\tan 17^{\circ}\end{align*}$

$\therefore x=17^{\circ}$
 

Attachments

  • Find MBC.JPG
    Find MBC.JPG
    8 KB · Views: 50
  • #15
johng said:
Albert, Opalg, someone please help. I still don't get it. In Albert's expression for angle BFG, the 77+77+60 is the angle AME, but from where comes 146?? Is he saying triangle AMB is isosceles?

The following sketch shows G' as the second tangency point from B and G the intersection of AM with the circle. I still fail to see why G = G'. (Of course the drawing is not accurate; the actual problem has G = G'.)

View attachment 1826
Yes, you are right again, and this time I don't see any way to fix it. The distorted diagram showing G and G' as distinct points highlights the difficulty in Albert's argument. Fortunately, anemone has now given us a watertight proof using Ceva's theorem.
 
  • #16

Attachments

  • Angle of MBC.jpg
    Angle of MBC.jpg
    36.3 KB · Views: 47
Last edited:
  • #17
Albert said:
View attachment 1837

hope this time you can understand it

Ah, I applaud your constant effort to try to show us the reasoning behind your work...but Albert, I only managed to follow up to where you mentioned why $G$ is the midpoint of $AM$, and I don't see how that would imply the two lines $BG$ and $AM$ are perpendicular to one another.

Also, if I could decipher that, what follows immediately is that the two triangles $BGA$ and $BGM$ are congruent triangles and hence $\angle ABM=180^{\circ}-2(73^{\circ})=34^{\circ}$ and that gives $\angle MBC=51^{\circ}-34^{\circ}=17^{\circ}$.
 
  • #18
anemone said:
Ah, I applaud your constant effort to try to show us the reasoning behind your work...but Albert, I only managed to follow up to where you mentioned why $G$ is the midpoint of $AM$, and I don't see how that would imply the two lines $BG$ and $AM$ are perpendicular to one another.

Also, if I could decipher that, what follows immediately is that the two triangles $BGA$ and $BGM$ are congruent triangles and hence $\angle ABM=180^{\circ}-2(73^{\circ})=34^{\circ}$ and that gives $\angle MBC=51^{\circ}-34^{\circ}=17^{\circ}$.
please see post #9
 
  • #19
Using your diagram and if it were up to me, I would continue, after I realized $G$ is the midpoint of $AM$, by using the trigonometric approach! Please pardon me because to prove something using only geometrical approach is my downfall. :eek:

My solution to determine the $\angle BGA=90^{\circ}$ would be as follow:

Since $AG=GM=r$, therefore we have $AM=2r$. If we consider the triangle $AMC$, using the Sine Rule we get $\dfrac{AC}{\sin (60+77)^{\circ}}=\dfrac{AM}{\sin 13^{\circ}}$, or, $AC=\dfrac{2r\sin 137^{\circ}}{\sin 13^{\circ}}$.

Again, by applying Sine Rule to the triangle $ABC$ gives

$\dfrac{AB}{\sin 16^{\circ}}=\dfrac{AC}{\sin 51^{\circ}}$, or, $AB=\dfrac{2r\sin 137^{\circ}\sin 26^{\circ}}{\sin 13^{\circ}\sin 51^{\circ}}$.

Next, we need to find the length of $BG$ and this could be done by applying Cosine Rule to the triangle $BAG$:

$BG^2=AG^2+AB^2-2(AG)(AB)\cos 73^{\circ}$

$\therefore BG=3.27085r$

Last, we could find the value for $\angle BGA$ by using the Sine Rule again:

$\dfrac{AB}{\sin \angle BGA}=\dfrac{BG}{\sin 73^{\circ}}$

This shows that $\angle BGA$ is indeed, $90^{\circ}$!(Tmi)(Sun)
 
  • #20
Albert said:
please see post #9
Post #9 states that $\angle FBG=\dfrac{77+77+60-146}{2}$. I believe that is the step that several of us find incomprehensible. Could you explain it please?
 
  • #21
Opalg said:
Post #9 states that $\angle FBG=\dfrac{77+77+60-146}{2}$. I believe that is the step that several of us find incomprehensible. Could you explain it please?
View attachment 1839
 

Attachments

  • Angle FBG.jpg
    Angle FBG.jpg
    18.8 KB · Views: 48
  • #22
I promise this is my last post to this thread. It is basically my post 5, but I am now happy with the proof. (Of course the figure as drawn is inaccurate.)

View attachment 1840
 

Attachments

  • MHBgeometry7b.png
    MHBgeometry7b.png
    13.2 KB · Views: 43

Related to Finding $\angle MBC$ in Triangle $ABC$ with Proof

1. How do you find the measure of angle MBC in Triangle ABC?

The measure of angle MBC in Triangle ABC can be found using the Law of Cosines or the Law of Sines. The Law of Cosines states that c^2 = a^2 + b^2 - 2abcosC, where c is the side opposite angle C. The Law of Sines states that sinA/a = sinB/b = sinC/c. By plugging in the known values for sides and angles, the measure of angle MBC can be calculated.

2. What is the proof for finding the measure of angle MBC in Triangle ABC?

The proof for finding the measure of angle MBC in Triangle ABC involves using the properties of triangles, such as the Triangle Sum Theorem and the Exterior Angle Theorem. By constructing a line parallel to side BC through point A, a triangle with known angles can be formed, allowing for the measure of angle MBC to be calculated using the Triangle Sum Theorem and the Exterior Angle Theorem.

3. What are the steps for finding the measure of angle MBC in Triangle ABC?

The steps for finding the measure of angle MBC in Triangle ABC involve first identifying the known sides and angles, then using the Law of Cosines or Law of Sines to calculate the measure of angle MBC. Alternatively, the proof method can be used by constructing a line parallel to side BC through point A, and then using the properties of triangles to calculate the measure of angle MBC.

4. Can the measure of angle MBC be found without using the Law of Cosines or Law of Sines?

Yes, the measure of angle MBC can also be found using the Triangle Sum Theorem and the Exterior Angle Theorem. By constructing a line parallel to side BC through point A, a triangle with known angles can be formed, allowing for the measure of angle MBC to be calculated using these theorems.

5. Are there any special cases where the measure of angle MBC cannot be determined?

Yes, there are special cases where the measure of angle MBC cannot be determined. One example is when Triangle ABC is a right triangle, as angle MBC would be the right angle and its measure would be 90 degrees. Another example is when there is not enough information given about the sides and angles of Triangle ABC to use the Law of Cosines, Law of Sines, or the proof method.

Similar threads

Replies
1
Views
964
Replies
1
Views
796
Replies
4
Views
847
Replies
1
Views
919
Replies
1
Views
811
Replies
5
Views
1K
  • General Math
Replies
4
Views
968
  • General Math
Replies
5
Views
1K
  • General Math
Replies
1
Views
1K
Replies
1
Views
836
Back
Top